Top Banner
Applied Technology Level 4 Worldwide Interactive Network, Inc. 1000 Waterford Place, Kingston, TN 37763 • 888.717.9461 ©2008 Worldwide Interactive Network, Inc. All rights reserved.
106

AT - 4 - Zohowinlearning.zohosites.com › files › PDF › Applied... · The use of materials in this manual does not imply any specific results when WIN materials are used with

Jul 07, 2020

Download

Documents

dariahiddleston
Welcome message from author
This document is posted to help you gain knowledge. Please leave a comment to let me know what you think about it! Share it to your friends and learn new things together.
Transcript
Page 1: AT - 4 - Zohowinlearning.zohosites.com › files › PDF › Applied... · The use of materials in this manual does not imply any specific results when WIN materials are used with

Applied TechnologyLevel 4

Worldwide Interactive Network, Inc.1000 Waterford Place, Kingston, TN 37763 • 888.717.9461

©2008 Worldwide Interactive Network, Inc. All rights reserved.

Page 2: AT - 4 - Zohowinlearning.zohosites.com › files › PDF › Applied... · The use of materials in this manual does not imply any specific results when WIN materials are used with

2 • Applied Technology

Copyright © 1998 by Worldwide Interactive Network, Inc. ALL RIGHTS RESERVED.Printed in the U.S.A. No part of this publication may be reproduced, stored in a retrievalsystem, or transmitted in any form or by any means, electronic, photocopying, recordingor otherwise without the prior written permission of Worldwide Interactive Network,Inc.

ACT™ and WorkKeys® are trademarks of ACT, Inc. Worldwide Interactive Network,Inc. is not owned or controlled by ACT, Inc.; however, ACT, Inc. has reviewed thesetraining materials and has determined that they meet ACT, Inc.’s standards for WorkKeysTraining curriculum. The WorkKeys employment system is a product of ACT, Inc.

The use of materials in this manual does not imply any specific results when WINmaterials are used with the ACT WorkKeys system.

Requests for permission to reproduce or make other use of this material should beaddressed to:

Worldwide Interactive Network, Inc.1000 Waterford PlaceKingston, Tennessee 37763 USATel: (865) 717-3333Fax: (865) [email protected]

Page 3: AT - 4 - Zohowinlearning.zohosites.com › files › PDF › Applied... · The use of materials in this manual does not imply any specific results when WIN materials are used with

Applied Technology • 3

INTRODUCTION

Hi! Let me introduce myself to you. I am EdWIN,and I will be your friendly guide through this study.You may have met me before in a different course or inLevel 3 of Applied Technology. If you have, I hope youenjoy learning with me. Look for me to pop up everynow and then with a tip, hint, or maybe even a quizquestion or two!

You are about to begin Applied Technology, Level 4.If you completed Level 3, you know that it wasn’t toodifficult. We will be studying thermodynamics, fluiddynamics, electricity, and mechanics in this course. Thislevel will review the basics. The main difference fromLevel 3 is that the exercises will be more difficult. Thegreat thing about this type of learning is that yougenerally can set your own pace. Consequently, youwill not have to move on until you feel you have learnedthe material to your satisfaction.

I am reviewing basics for two reasons: (1) you canreview and reinforce basic principles at each new level,and (2) new participants won’t miss basic information.

So, if you are ready, so am I. Turn the page and let’sbegin!

Hi, I’m EdWIN!

Page 4: AT - 4 - Zohowinlearning.zohosites.com › files › PDF › Applied... · The use of materials in this manual does not imply any specific results when WIN materials are used with

4 • Applied Technology

Problem-Solving Strategies

The basic components of effective problem-solving strategies are –

• Identifying the problem (“What is the goal?” and “What limits does the goalimpose?”)

• Analyzing and interpreting data (reading a gauge, interpreting a printout).This includes identifying and disregarding nonessential data.

• Exploring and evaluating solutions (“What options are available?” and “Whichoption is best, taking into account many variables, including cost, time, humanresources, materials, environment, and expertise?”)

One well-known problem-solving model is the IDEAL* model. The IDEALmodel was designed as an aid for teaching and improving problem-solving skills.The IDEAL process includes the following steps:

I = Identify the problem (determine what needs to be done).D = Define and represent the problem (sharpen and clarify the boundaries).E = Explore alternative approaches (analyze and evaluate alternatives).A = Act on a plan (determine the logical steps to be used and how to progress

through the steps).L = Look at the result (determine whether or not the plan worked).

The exercises in this workbook guide learners through the IDEAL problem-solving process.

* The IDEAL Problem Solver: A Guide for Improving Thinking, Learning, and Creativity , © 1984. Permission for use granted by W.H. Freeman and Company/Worth Publishers, all rights reserved.

STRATEGIES

Page 5: AT - 4 - Zohowinlearning.zohosites.com › files › PDF › Applied... · The use of materials in this manual does not imply any specific results when WIN materials are used with

Applied Technology • 5

Problem-solving strategies are critical to any instruction aimed at improving alllevels of applied technology skills. To be effective technological problem solvers,you need to be able to do the following:

• Understand cause-effect relationships (What parts of systems affect and areaffected by other parts?)

• Make comparisons (What commonalities and differences do systems have?)• Recognize probable outcomes (How will the system react to a specific action?)• Predict what should happen next (Based on what has been observed, what is

known about a specific system, and what is known about related scientificprinciples, make a prediction about what will happen next.)

• Judge spatial relationships (Visualize how a system operates and mentally rotatesystem parts to solve problems within a given system.)

• Notice what appears out of place (Observe a malfunctioning system in operationto determine what is not working correctly.)

Course Strategies

The best way for me to help you be an effective problem solver is to give youopportunities to develop and refine your problem-solving skills. Therefore, I willuse the following strategies:

• I will minimize instructions so that you are encouraged to invent innovativeways to accomplish the tasks.

• I will provide you with a variety of materials from which to choose.• If you have questions about the exercises, reread the beginning instructions.

This strategy encourages you to figure out how to use the materials to reachyour goal.

• You have plenty of time to explore. As long as you are actively engaged, learningis taking place. If ample time is allowed, you will be able to do more in-depthinvestigation. Thinking about a problem is part of learning.

• I encourage you to share ideas with others. This strategy reflects how peoplesolve problems in the workplace — with input from others. Most problem-solving activities lend themselves to having you work individually, but you areencouraged to seek others’ input.

Let’s review some basic scientific principles

STRATEGIES

Page 6: AT - 4 - Zohowinlearning.zohosites.com › files › PDF › Applied... · The use of materials in this manual does not imply any specific results when WIN materials are used with

6 • Applied Technology

concerning Applied Technology that we covered in thefirst level. Applied Technology focuses on:

• Principles related to power sources – for thermal,fluid, electrical, and mechanical systems.

• Principles related to flow – for thermal, fluid,electrical, and mechanical systems.

• Principles related to pressure – for thermal, fluid,electrical, and mechanical systems.

• Principles related to resistance – for thermal, fluid,electrical, and mechanical systems.

Some basic scientific principles involved with energysources, flow, pressure, and resistance follow:

Bernoulli’s principle: The faster the flow of air or fluid, the lower the pressure.

Boyle’s law: For a certain amount of gas, at a constant temperature, as the pressure(P) increases, the volume (V) of the gas decreases so that P times V is constant (k).(PV=k).

Charles’ law: For a certain amount of gas, at a constant pressure, as the absolutetemperature of the gas increases, the volume of the gas also increases.Mathematically this is: Volume (V) divided by temperature (T) equals a constant(k). V/T=k The temperature must be on an absolute scale that is in reference toabsolute zero.

Hooke’s law: The greater the force exerted on an object, the more it will be moved.For example, the heavier the weight hanging from a spring, the more the springwill be stretched.

Newton’s laws of motion:• An object will remain at rest or in uniform motion unless acted upon by an

outside force.• When a force acts upon an object, it changes the momentum of that object,

and this change is proportional to the applied force and to the time that it actsupon the object.

• Every action (force) is followed by an equal and opposite reaction (force).

Ohm’s law: Current is directly proportional to the voltage and inverselyproportional to the resistance.

Pascal’s law: Pressure added to a confined fluid at any point instantly appearsequally at all other points, and is always at right angles to the containing surfaces.

BASIC PRINCIPLES

Page 7: AT - 4 - Zohowinlearning.zohosites.com › files › PDF › Applied... · The use of materials in this manual does not imply any specific results when WIN materials are used with

Applied Technology • 7

LESSON 1 Thermodynamics

LESSON 2 Fluid Dynamics

LESSON 3 Electricity

LESSON 4 Mechanics

LESSON 5 Posttest

REFERENCES Test-Taking TipsBasic Scientific PrinciplesBibliographyAnswers to Pop Quiz Questions

OUTLINE

Page 8: AT - 4 - Zohowinlearning.zohosites.com › files › PDF › Applied... · The use of materials in this manual does not imply any specific results when WIN materials are used with

8 • Applied Technology

THERMODYNAMICS

Thermodynamics is a pretty complicated soundingword, isn’t it? There is a lot that can be learned aboutit. But for our purposes, we are not going to try tobecome experts in the subject! We are going to learn,however, the basic concept of the topic and learn tosolve some basic problems using our acquiredknowledge.

All matter is made up of particles that are in constantmotion. This motion manifests itself as the form ofenergy called heat. The study of heat is calledthermodynamics. Now, that wasn’t so hard, was it?There are two basic laws of thermodynamics.

Basic Laws of Thermodynamics

1. Energy cannot be created or destroyed.

2. Heat energy always flows spontaneously fromhot to cold.

Reflection and absorption

In this level we will be discussing the basic conceptsof heat and absorption. Heat is a form of energy and ismeasured by the amount of work done. Absorption isthe amount of heat that can be taken in by anyparticular object and not reflected back or away fromthe object.

LESSON 1

Absorbing solar heat!

Page 9: AT - 4 - Zohowinlearning.zohosites.com › files › PDF › Applied... · The use of materials in this manual does not imply any specific results when WIN materials are used with

Applied Technology • 9

For example, have you ever noticed or wonderedwhy your concrete driveway is still covered in snow orice long after the asphalt roads were clear? Well,absorption is the answer. The light surface of concretereflects the heat of the sun while the dark asphaltabsorbs the heat. Since more heat is absorbed by theasphalt, it is only reasonable that the snow and icewould melt from the dark surface quicker than fromthe light surface of the concrete.

This is also why people wear light or white colorsin the summer as opposed to dark. Conversely, theywear dark colors in the winter to absorb as much heatas possible.

This same principle also applies to rough or dullsurfaces as opposed to smooth, shiny surfaces. In arough surface, the dips and undulations of the surfacewill absorb more heat. The smooth surface does notcontain these dips and therefore, reflects more heat awayfrom itself. To understand these concepts more clearly,study the following illustrations.

LESSON 1

Page 10: AT - 4 - Zohowinlearning.zohosites.com › files › PDF › Applied... · The use of materials in this manual does not imply any specific results when WIN materials are used with

10 • Applied Technology

Once again, this principle holds true with differentcolors, shades, and textures of material. A gray surfacewill reflect more heat than a darker gray, and acompletely smooth surface will reflect more than onethat is slightly rougher. Which would absorb more heat;dark brown shingles or light gray shingles? Both surfacesare of the same rough texture, but since the darker colorwould absorb more, then the brown shingles wouldabsorb more heat. Which would reflect more heat; awhite silk shirt or a white cotton one? Since the colorof the shirts is the same but the texture is different, thesilk shirt would reflect more heat because it is thesmoother of the two.

Some other generalizations that can be made aboutheat include that heat travels through conductors suchas metal better than it travels through insulators suchas brick or wood. For example, you know that if youstir boiling water with a metal spoon, it will soon heatup to the point where you cannot touch it. If you stirboiling water with a wooden spoon, you can stir itindefinitely without it getting too hot to touch.

LESSON 1

Pop Quiz:Is my club a conductor oran insulator?

Page 11: AT - 4 - Zohowinlearning.zohosites.com › files › PDF › Applied... · The use of materials in this manual does not imply any specific results when WIN materials are used with

Applied Technology • 11

Friction also causes heat. When friction causes heat,the object that is in constant contact gets hotter thanthe movable object.

For example, do you recall the technique of startinga fire using two sticks and twine? The twine is wrappedaround the vertical stick and the horizontal stick isinserted into the twine that is wrapped around thevertical stick. By moving the horizontal stick to makethe vertical one move back and forth, the tinder willeventually become hot enough to burst into flame. Whydoesn’t the vertical stick catch fire also? Because thevertical stick is the movable object, and the tinder isthe stationary object. Of course, you must be verypatient because it takes a long time to start a fire thisway. (That’s why matches were invented, I’m sure!)

The same will hold true for sawing wood. The woodwill get hotter than the saw blade because it is inconstant contact with the friction caused by the blade,which is the movable object. Of course, this does notmean that the saw blade will not heat up at all. In fact,if you have ever touched the blade after sawing, youprobably noticed that it was warm to the touch. Frictioncaused this heat also.

In this level of Applied Technology, we willconcentrate on some vocabulary concerning thermo-dynamics. Then, we will complete some exercises usingthe principles we have learned.

LESSON 1

Page 12: AT - 4 - Zohowinlearning.zohosites.com › files › PDF › Applied... · The use of materials in this manual does not imply any specific results when WIN materials are used with

12 • Applied Technology

Concerning conduction

Conduction is defined as the passage of energy (heatand electricity) from particle to particle. Some objectsconduct heat better than others. Metal is the mostcommon substance used when you want to conductheat (or electricity, for that matter). Depending on theapplication, you may want to use a substance thatconducts well, or you may want to choose somethingthat does not conduct as well. For example, if you wantto make pudding on the stove, you want to stir it withsomething that does not conduct heat well. Sincepudding requires constant stirring during part of thepreparation, you would not want to use a metal spoon,but a wooden one. This way you can stir constantlywithout the spoon heating up and burning you as youstir.

The same is true for other applications. Why areoutdoor grills always made of metal? The grill surfaceneeds to conduct heat to the hamburgers that are placedupon it. If the grill surface were made of somethingthat was not only fireproof, but also nonconductive,your hamburgers would take forever to get done. You’regoing to be waiting a long time for lunch!

OK, let’s consider the exercises using this principle.

LESSON 1

Page 13: AT - 4 - Zohowinlearning.zohosites.com › files › PDF › Applied... · The use of materials in this manual does not imply any specific results when WIN materials are used with

Applied Technology • 13

EXERCISE – TEMPERATURE

Instructions: Look at the following diagram and answer the questions related to it.

1. What happens to the temperature of the spoon when it is placed inthe hot chocolate?

____________________________________________________________

____________________________________________________________

2. Do you think the spoon is hotter at the bottom, at the top, or is it thesame throughout the length of the spoon? ____________________

3. Will the spoon affect the cooling time of the hot chocolate? If so,how?

____________________________________________________________

____________________________________________________________

LESSON 1

Page 14: AT - 4 - Zohowinlearning.zohosites.com › files › PDF › Applied... · The use of materials in this manual does not imply any specific results when WIN materials are used with

14 • Applied Technology

ANSWERS TO EXERCISE

1. What happens to the temperature of the spoon when it is placed inthe hot chocolate?

Answer: The spoon will heat up.

2. Do you think the spoon is hotter at the bottom, at the top, or is it thesame throughout the length of the spoon?

Answer: The heat at the bottom of the spoon will be hotter. The heatwill be conducted up the handle; however, the temperature atthe top of the spoon will not reach the level of the bottomsince the liquid is cooling.

3. Will the spoon affect the cooling time of the hot chocolate? If so,how?

Answer: The hot chocolate will cool faster because the spoon createsa larger surface area to dissipate the heat.

LESSON 1

Page 15: AT - 4 - Zohowinlearning.zohosites.com › files › PDF › Applied... · The use of materials in this manual does not imply any specific results when WIN materials are used with

Applied Technology • 15

EXERCISE – REFLECTION AND ABSORPTION

Instructions: Read the scenario concerning this principle of heat and absorption and answerquestions relating to it.

Scenario

You want to leave a water dish outside for your pet on a coldbut sunny day. Since you will be gone all day, you want toinsure that your pet’s water will not freeze.

1. What type of container would be best to use since it is a sunny day?

a. metalb. plasticc. pottery

2. What color would you choose for the container?

a. blackb. whitec. gray

3. Where would you place the container?

a. shielded locationb. direct sunlightc. shady area

4. What would be your explanation for the choices you made?

____________________________________________________________

____________________________________________________________

LESSON 1

Page 16: AT - 4 - Zohowinlearning.zohosites.com › files › PDF › Applied... · The use of materials in this manual does not imply any specific results when WIN materials are used with

16 • Applied Technology

ANSWERS TO EXERCISE

1. What type of container would be best to use since it is a sunny day?

Answer: a. metal

2. What color would you choose for the container?

Answer: a. black

3. Where would you place the container?

Answer: b. direct sunlight

4. What would be your explanation for the choices you made?

Answer: Since it is a sunny day, I would choose a black metal panbecause metal conducts heat better and black absorbs moreheat. I would choose a location where the pan would get directsunlight for as much of the day as possible.

LESSON 1

Page 17: AT - 4 - Zohowinlearning.zohosites.com › files › PDF › Applied... · The use of materials in this manual does not imply any specific results when WIN materials are used with

Applied Technology • 17

<

Heat transfer

Heat can be transferred from one object to another.As in the previous scenario, we wanted to choose thebest possible way of keeping the water from freezing inthe pan during the day. We chose a material that wouldtransfer heat through conduction and a color thatwould absorb as much heat as possible to be transferred.

Let’s look at another example of heat transfer. I willdescribe a scenario and ask you to make a predictionabout what would happen.

Thinking Activity

Place a full soft drink can (at room temperature) in a pie pan, cover it with a clayflower pot. Pour water over the flower pot, soaking it, and pour about an inch ofwater in the pan. Place it in direct sunlight.

What do you think will happen? Take time to thinkabout it. You may want to actually conduct anexperiment.

LESSON 1

Page 18: AT - 4 - Zohowinlearning.zohosites.com › files › PDF › Applied... · The use of materials in this manual does not imply any specific results when WIN materials are used with

18 • Applied Technology

This one might surprise you. As the waterevaporates, its vapor will carry heat away from the claypot. This cools the air in the clay pot, which cools thesoft drink. In about an hour, the soda should be coolenough to drink.

LESSON 1

Page 19: AT - 4 - Zohowinlearning.zohosites.com › files › PDF › Applied... · The use of materials in this manual does not imply any specific results when WIN materials are used with

Applied Technology • 19

EXERCISE – VOCABULARY

Instructions: Match each term in the left column with its definition in the right column bywriting the letter which represents the appropriate definition in the spaceprovided. You can check your answers on the next page.

___ absorption a. a utensil for flipping food over to the otherside

___ conduction b. exchanging one substance for another

___ conductor c. capable of transmitting heat

___ molecules d. a tool

___ reflection e. related to heat

___ spatula f. a degree of hotness or coldness that ismeasured on a scale

___ temperature g. turning back

___ thermal h. the smallest particle of a substance thatretains all the properties of the substance

___ transfer i. interception of radiant energy

___ utensil j. movement of heat through a solid object

LESSON 1

Page 20: AT - 4 - Zohowinlearning.zohosites.com › files › PDF › Applied... · The use of materials in this manual does not imply any specific results when WIN materials are used with

20 • Applied Technology

ANSWERS TO EXERCISE

Now that we have covered the basics ofthermodynamics, thought about a few scenarios, andbrushed up on our vocabulary, let’s try a few moreexercises.

i absorption a. a utensil for flipping food over to the otherside

j conduction b. exchanging one substance for another

c conductor c. capable of transmitting heat

h molecules d. a tool

g reflection e. related to heat

a spatula f. a degree of hotness or coldness that ismeasured on a scale

f temperature g. turning back

e thermal h. the smallest particle of a substance thatretains all the properties of the substance

b transfer i. interception of radiant energy

d utensil j. movement of heat through a solid object

LESSON 1

Page 21: AT - 4 - Zohowinlearning.zohosites.com › files › PDF › Applied... · The use of materials in this manual does not imply any specific results when WIN materials are used with

Applied Technology • 21

EXERCISE – PICNIC

Instructions: Read the following scenario and answer the questions using your knowledge ofthermodynamics.

Scenario

You are going on an outdoor picnic on a hot, sunny day. Youare asked to bring a spatula for the grill and an ice chest. Youhave several spatulas and ice chests at home. You mustchoose which would be the best to bring to the picnic, fromthe following choices:

Spatulas: all metal, all wood, metal with a wooden handle,black plastic

Ice chests: white, black

Identify the problem

1. What is your assignment for the picnic?

a. Determine how much meat you will need to buy for 8 people.b. Check the weather forecast for the day of the picnic.c. Select the most appropriate spatula and ice chest to take to the

picnic.d. Count the number of people who will attend the picnic.e. Buy block ice or chipped ice.

LESSON 1

Page 22: AT - 4 - Zohowinlearning.zohosites.com › files › PDF › Applied... · The use of materials in this manual does not imply any specific results when WIN materials are used with

22 • Applied Technology

Define the problem

2. What do you need to consider when selecting items for a picnic ona sunny day?

a. outside temperature and type of items neededb. outside temperature and how many people will attend the picnicc. how many people will be at the picnicd. how many pounds of meat to buye. the name of the park where the picnic will take place

Explore alternatives

It is a hot, sunny day and the picnic is outdoors. Answer the following questions to determinewhether you should take metal, wood, metal with a wooden handle, or plastic utensils andwhether you should take a black or white cooler.

3. If you select the all-wooden spatula, what will most likely happenwhen you grill?

a. The spatula might catch fire.b. The spatula will burn your hand.c. The spatula will lose its form.d. Food will stick to the spatula.e. The spatula will lose its strength and break.

4. If you select the all-metal spatula, what will most likely happen whenyou grill?

a. The spatula will transfer heat to your hand and burn it.b. The spatula will absorb juices from the meat.c. The spatula will stick to the meat.d. The spatula will add a metal taste to the meat.e. The spatula will burn and lose its shape.

LESSON 1

Page 23: AT - 4 - Zohowinlearning.zohosites.com › files › PDF › Applied... · The use of materials in this manual does not imply any specific results when WIN materials are used with

Applied Technology • 23

IDEAL

5. If you select the plastic spatula, what will most likely happen whenyou grill?

a. The spatula will burn your hand.b. The spatula will not lose its shape.c. The spatula will become brittle and break.d. The spatula will be OK if you quench it with water.e. The spatula will melt and lose its shape.

6. If you select the metal spatula with a wooden handle, what will mostlikely happen when you grill?

a. The spatula will remain sturdy and will not conduct heat to yourhand.

b. The handle will separate from the metal.c. The spatula will give a metal taste to the meat.d. Food will stick to the spatula.e. The spatula will burn up.

7. If you select the black cooler for the picnic, what will most likelyhappen?

a. Food will stay cold in the black cooler.b. The black cooler will reflect heat, and ice will not melt.c. Ants can get into either the black or white cooler.d. The inside temperature of the black cooler will remain stable.e. The black cooler will absorb heat and melt the ice within it more

quickly than a light-colored cooler would.

LESSON 1

Page 24: AT - 4 - Zohowinlearning.zohosites.com › files › PDF › Applied... · The use of materials in this manual does not imply any specific results when WIN materials are used with

24 • Applied Technology

8. If you select the white cooler for the picnic, what will most likelyhappen?

a. The white cooler will stay clean.b. The white cooler will reflect heat, and the ice will stay longer than

in a black cooler.c. Bees will be attracted to the white cooler.d. The white cooler will be easier to carry than the black cooler.e. The white cooler will absorb heat quickly and will cause the ice

to melt.

Act on a plan

9. Select the most appropriate supplies to bring to the picnic.

a. Bring the all-wooden utensil and a black cooler.b. Bring the all-metal utensil and a white cooler.c. Bring the plastic utensil and a black cooler.d. Bring the metal spatula with the wooden handle and the white

cooler.e. Purchase a new cooler and a new spatula.

Look at the result

10. You take the metal spatula with the wooden handle and the whitecooler to the picnic. What are the effects or results of this choice?

a. You were able to flip all the hamburgers successfully and thecooler stayed cool.

b. You burned your hand, but the cooler stayed cool.c. The spatula melted and so did the ice in the cooler.d. The spatula caught fire and burned up, and the white cooler got

dirty.e. The spatula became brittle and broke, and the cooler leaked.

LESSON 1

Page 25: AT - 4 - Zohowinlearning.zohosites.com › files › PDF › Applied... · The use of materials in this manual does not imply any specific results when WIN materials are used with

Applied Technology • 25

ANSWERS TO EXERCISE

1. What is your assignment for the picnic?

Answer: c. Select the most appropriate spatula and ice chest to taketo the picnic.

2. What do you need to consider when selecting items for a picnic ona sunny day?

Answer: a. outside temperature and type of items needed

3. If you select the all-wooden spatula, what will most likely happenwhen you grill?

Answer: a. The spatula might catch fire.

4. If you select the all-metal spatula, what will most likely happen whenyou grill?

Answer: a. The spatula will transfer heat to your hand and burn it.

5. If you select the plastic spatula, what will most likely happen whenyou grill?

Answer: e. The spatula will melt and lose its shape.

6. If you select the metal spatula with a wooden handle, what will mostlikely happen when you grill?

Answer: a. The spatula will remain sturdy and will not conduct heat toyour hand.

LESSON 1

Page 26: AT - 4 - Zohowinlearning.zohosites.com › files › PDF › Applied... · The use of materials in this manual does not imply any specific results when WIN materials are used with

26 • Applied Technology

7. If you select the black cooler for the picnic, what will most likelyhappen?

Answer: e. The black cooler will absorb heat and melt the ice within itmore quickly than a light-colored cooler would.

8. If you select the white cooler for the picnic, what will most likelyhappen?

Answer: b. The white cooler will reflect heat, and the ice will stay longerthan in a black cooler.

9. Select the most appropriate supplies to bring to the picnic.

Answer: d. Bring the metal spatula with the wooden handle and thewhite cooler.

10. You take the metal spatula with the wooden handle and the whitecooler to the picnic. What are the effects or results of this choice?

Answer: a. You were able to flip all the hamburgers successfully andthe cooler stayed cool.

LESSON 1

Page 27: AT - 4 - Zohowinlearning.zohosites.com › files › PDF › Applied... · The use of materials in this manual does not imply any specific results when WIN materials are used with

Applied Technology • 27

Friction

We have discussed friction earlier in this course.Friction causes heat to be generated when systemsoperate. Friction can be used to our advantage in someinstances, such as starting a fire. But in many instances,friction is the enemy. Consider the reason you keep oilin your car. What happens if you don’t? Yes, you willbe buying a new engine … right? That’s because thefriction caused by the up-and-down motion of thepistons will literally burn up an engine without oilwhich limits friction. Friction can be reduced byproviding lubrication to the system.

Let’s try an exercise related to the principle of friction.

LESSON 1

Ready to create friction …

Page 28: AT - 4 - Zohowinlearning.zohosites.com › files › PDF › Applied... · The use of materials in this manual does not imply any specific results when WIN materials are used with

28 • Applied Technology

EXERCISE – MACHINE SHOP BEARINGS AND SHAFT

Instructions: Read the following scenario and answer the questions using your knowledge offriction.

Scenario

While running a machine, you notice that the bronze shaftgets hot at the same time every day. You also notice a pile ofshavings under the stainless steel sleeve bearing every day.It is your job to determine why the shaft is overheating.

Identify the problem

1. What is your job?

a. to determine whether the belt is slippingb. to determine whether the bearing is too large for the shaftc. to check to see if the shaft is overheatingd. to check to see if the shaft is made of nickel alloye. to check the temperature of the machine

Define the problem

2. To begin to solve the problem, it would be BEST to:

a. check the rpm (revolutions per minute) on the shaft.b. check to see if the bearing is lubricated.c. disassemble the machine to compare the bearing’s diameter to

the dimensions noted for items on the parts list.d. check the v-belt width against the specifications on the parts list.e. measure the air temperature near the machine.

LESSON 1

Page 29: AT - 4 - Zohowinlearning.zohosites.com › files › PDF › Applied... · The use of materials in this manual does not imply any specific results when WIN materials are used with

Applied Technology • 29

Explore alternatives

3. Which plan of action would BEST determine the cause of the shaftoverheating?

a. If the shaft is too hot, check to see if the machine is close to theheater.

b. If the belt is not hot, check to see if the shavings are of the samematerial as the flywheel.

c. If the shaft and the flywheel are hot and the belt is cool, the beltcould be slipping.

d. If the shaft and the bearings are hot, check to see what materialthe shavings are made of.

e. If the shaft is too hot, check to see if the belt is broken.

Act on a plan

4. The next logical step to a more permanent solution would be to:

a. tighten the belt.b. replace the belt.c. replace and lubricate the bearings.d. lubricate the existing bearing and start up the machine again.e. lubricate the belt.

Look at the result

5. To see if the problem has been solved, you should restart themachine and:

a. check to see if the shaft and bearings stay cool.b. listen for squeaky belts.c. listen for a rattling flywheel.d. check to see if the temperature of the belt stays cool.e. check the temperature of the room near the machine.

LESSON 1

Page 30: AT - 4 - Zohowinlearning.zohosites.com › files › PDF › Applied... · The use of materials in this manual does not imply any specific results when WIN materials are used with

30 • Applied Technology

ANSWERS TO EXERCISE

1. What is your job?

Answer: c. to check to see if the shaft is overheating

2. To begin to solve the problem, it would be BEST to:

Answer: b. check to see if the bearing is lubricated.

3. Which plan of action would BEST determine the cause of the shaftoverheating?

Answer: d. If the shaft and the bearings are hot, check to see whatmaterial the shavings are made of.

4. The next logical step to a more permanent solution would be to:

Answer: c. replace and lubricate the bearings.

5. To see if the problem has been solved, you should restart themachine and:

Answer: a. check to see if the shaft and bearings stay cool.

LESSON 1

Page 31: AT - 4 - Zohowinlearning.zohosites.com › files › PDF › Applied... · The use of materials in this manual does not imply any specific results when WIN materials are used with

Applied Technology • 31

FLUID DYNAMICS

We will be concentrating on the basic principles offluid dynamics in this lesson. In Level 3 of AppliedTechnology, we discussed the relationship between fluiddepth and pressure. In this level we will discuss therelationship between fluid flow and resistance.

As resistance increases, fluid flow decreases.

I have chosen to discuss this principle because thereare many occupations that apply this principle in theworkplace. Some of them are firefighting, medical andintravenous drips, irrigation, application of pesticidesand insecticides, cooling systems, paint sprayingsystems, welding systems, and drinking water systems.Perhaps you can think of others that I did not mention.However, you can see that there are many workplaceapplications, and understanding this principle isimportant.

LESSON 2

The workplace is not theonly place it is important

to understand fluiddynamics.

Page 32: AT - 4 - Zohowinlearning.zohosites.com › files › PDF › Applied... · The use of materials in this manual does not imply any specific results when WIN materials are used with

32 • Applied Technology

<

Let’s consider an experiment to clarify this concept.

Thinking Activity

Assume you have a portable camping shower, several differentdiameters of aquarium hose, a 5-gallon bucket to catch water,cotton balls, and a stopwatch to measure the time of flow.

Imagine that you hook up the hose with the largest diameterto the shower … then the next largest, and so forth.

Which hose size will cause the water to flow throughthe quickest?

The hose with the largest diameter.

Why?

There is less resistance to the flow.

LESSON 2

Page 33: AT - 4 - Zohowinlearning.zohosites.com › files › PDF › Applied... · The use of materials in this manual does not imply any specific results when WIN materials are used with

Applied Technology • 33

Now, imagine doing the same thing except put a kink in eachhose.

What do you think will be the result?

Assuming the kinks are the same, the larger hose will stillfill the bucket fastest.

What would happen if you put a cotton ball in thehose?

You would increase the resistance, causing the bucket tofill up at a slower rate.

Now, I am sure you had little or no trouble thinkingabout this scenario. It is pretty basic. I have no doubtthat you already knew that this would happen even ifyou did not know the scientific principle behind it!

Let’s use the IDEAL strategy to consider the nextexercise.

LESSON 2

Pop Quiz:Define resistance.

Page 34: AT - 4 - Zohowinlearning.zohosites.com › files › PDF › Applied... · The use of materials in this manual does not imply any specific results when WIN materials are used with

34 • Applied Technology

EXERCISE – GARDEN WATER HOSE

Instructions: Read the following scenario and answer the questions using your knowledge offluid dynamics.

Scenario

You are asked to water the garden at the back of the building.The 50-foot, 3/4”-diameter hose you have is too short. Youadded a section of 50-foot, 1/2”-diameter hose with a coupler.You then adjusted the faucet at the house so that it is fullyopen. No water flows from the hose.

Identify the problem

1. What is your assignment?

a. to add another length of hoseb. to decide on the amount of water needed for the gardenc. to get a larger diameter hosed. to restore water flow to enable you to water the gardene. to measure the total length of hose needed to reach the garden

Define the problem

2. What is the FIRST step to gain water flow?

a. Call the plumber.b. Find a cause for the lack of water flow.c. Add more hose.d. Remove a hose.e. Turn off the valve.

LESSON 2

Page 35: AT - 4 - Zohowinlearning.zohosites.com › files › PDF › Applied... · The use of materials in this manual does not imply any specific results when WIN materials are used with

Applied Technology • 35

Explore alternatives

3. Which of the following would NOT be a possible cause for the lackof water flow?

a. defective nozzleb. kinks in a hosec. dirt or other debris in one of the hosesd. a large hole in one of the hosese. the large diameter of the hose

Act on a plan

4. What step should be taken FIRST?

a. Replace the hose.b. Look for obstructions.c. Adjust the water faucet.d. Call the plumber.e. Elevate the hose.

Look at the result

5. You found a kink in the hose and straightened it. What should beyour NEXT step?

a. Squeeze the nozzle.b. Adjust the faucet.c. Pay the plumber.d. Disconnect the hose.e. Check the connections for tightness.

LESSON 2

Page 36: AT - 4 - Zohowinlearning.zohosites.com › files › PDF › Applied... · The use of materials in this manual does not imply any specific results when WIN materials are used with

36 • Applied Technology

ANSWERS TO EXERCISE

1. What is your assignment?

Answer: d. to restore water flow to enable you to water the garden

2. What is the FIRST step to gain water flow?

Answer: b. Find a cause for the lack of water flow.

3. Which of the following would NOT be a possible cause for the lackof water flow?

Answer: e. the large diameter of the hose

4. What step should be taken FIRST?

Answer: b. Look for obstructions.

5. You found a kink in the hose and straightened it. What should beyour NEXT step?

Answer: a. Squeeze the nozzle.

LESSON 2

Page 37: AT - 4 - Zohowinlearning.zohosites.com › files › PDF › Applied... · The use of materials in this manual does not imply any specific results when WIN materials are used with

Applied Technology • 37

<

The next exercise will deal with Pascal’s law. Pascal’slaw states that pressure applied to an enclosed fluid istransmitted equally in all directions to every portion ofthe fluid and to the walls of the container.

For example, if you have a water balloon lying on aflat surface and you set an object (perhaps a full sodacan) on it, the pressure that goes in all directions againstthe walls of the balloon will be the same. Of course,too much pressure and you’ll likely get wet!

Consider the following experiment to illustrate thisprinciple:

Thinking Activity

Assume you have a three-foot rubber hose, water, syringe, and pressuregauge. This is an experiment that will demonstrate how a basic hydraulicbrake system works.

Place the pressure gauge at one end of the tube; fill the tube with water;and, place the syringe on the other end of the tube. Now, compress thesyringe. What do you think will happen? The air that is being forced intothe tube by the compression of the syringe will be indicated on the pressuregauge at the end of the tube. When released, the pressure gauge will goback to a zero setting.

Assume you have one hose that branches and connects to two pressuregauges. When you compress the syringe, air pressure will flow down bothtubes to the pressure gauges. The gauges will read the same pressure becausethe same pressure is being applied at the top of the tube.

Let’s try an exercise using this principle.

LESSON 2

Page 38: AT - 4 - Zohowinlearning.zohosites.com › files › PDF › Applied... · The use of materials in this manual does not imply any specific results when WIN materials are used with

38 • Applied Technology

EXERCISE – CAR BRAKES

Instructions: Using the following scenario and diagram, answer the questions related toPascal’s law.

Scenario

You are driving your car north on an interstate highway at aspeed of 57 miles per hour. As you prepare to exit, you depressthe brake pedal to slow the car down. The pedal goes to thefloor, and the car does not slow down. You steer to the bermas you quickly pump the brakes 8-10 times, and the cargradually begins to slow down. To stop the vehicle, you applythe emergency brake and the car eventually stops. What willyou do to solve the problem before getting on the road again?Refer to the diagram to answer the questions that follow.

LESSON 2

Braking System

Page 39: AT - 4 - Zohowinlearning.zohosites.com › files › PDF › Applied... · The use of materials in this manual does not imply any specific results when WIN materials are used with

Applied Technology • 39

Identify the problem

1. What is the problem?

a. The brake pedal is too small to stop the movement of the car.b. You were not driving at the posted speed limit.c. You must have taken the wrong exit.d. The brakes were not working properly.e. The road was too wet.

Define the problem

2. Why did the car eventually stop?

a. It ran out of gas.b. Although the hydraulic brake system failed, the emergency brake

worked.c. The hydraulic brake system was in working order, but the

emergency brakes failed.d. The car was going uphill.e. The car needs to use two braking systems instead of one in order

to stop.

Explore alternatives

3. Which system is most likely to have failed, preventing the car fromstopping when the brake pedal was applied?

a. brake shoesb. wheelsc. brake drumd. brake fluide. battery

LESSON 2

Page 40: AT - 4 - Zohowinlearning.zohosites.com › files › PDF › Applied... · The use of materials in this manual does not imply any specific results when WIN materials are used with

40 • Applied Technology

4. Although the brake pedal was pumped, the vehicle did not stop asexpected. Why would pumping the brake pedal begin the process ofslowing the vehicle, but not cause it to stop as normally expected?

a. The tires in the vehicle are worn and need to be replaced.b. The road conditions do not allow for proper stopping of vehicles.c. Pumping increased pressure; however, pressure could not be

maintained.d. The brake pedal is an incorrect size for the size of the vehicle.e. You lack the strength to properly press on the brake pedal initially,

causing a need for pumping.

Act on a plan

5. For what reason is it important to know where the leak in a hydraulicbrake system is located?

a. You must determine which parts to order from the parts manager.b. Unless you determine the source of the fluid escape, you cannot

restore the system to its properly closed function.c. Brake fluid might collect in places that are not the source of the

leak.d. All of the above are correct.e. None of the above are correct.

LESSON 2

Page 41: AT - 4 - Zohowinlearning.zohosites.com › files › PDF › Applied... · The use of materials in this manual does not imply any specific results when WIN materials are used with

Applied Technology • 41

Look at the result

6. The leak has been discovered and repaired in a malfunctioninghydraulic brake system. Before driving the vehicle, what should takeplace?

a. The vehicle should be thoroughly washed to remove any brakefluid from the car’s finish.

b. The seat should be readjusted to compensate for the change inbrake pedal position, if any.

c. With the wheels removed, have pressure applied to the brakepedal while observing the brake shoes and pads to ensure theirproper activation.

d. Drive the vehicle at a slow speed in an open area to be sure itisn’t damaged.

e. With the wheels removed, drive the vehicle at a slow speed in anopen area while observing the brake shoes and pads to ensuretheir proper activation.

IDEAL

LESSON 2

Page 42: AT - 4 - Zohowinlearning.zohosites.com › files › PDF › Applied... · The use of materials in this manual does not imply any specific results when WIN materials are used with

42 • Applied Technology

ANSWERS TO EXERCISE

1. What is the problem?

Answer: d. The brakes were not working properly.

2. Why did the car eventually stop?

Answer: b. Although the hydraulic brake system failed, the emergencybrake worked.

3. Which system is most likely to have failed, preventing the car fromstopping when the brake pedal was applied?

Answer: d. brake fluid

4. Although the brake pedal was pumped, the vehicle did not stop asexpected. Why would pumping the brake pedal begin the process ofslowing the vehicle, but not cause it to stop normally as expected?

Answer: c. Pumping increased pressure; however, pressure could notbe maintained.

5. For what reason is it important to know where the leak in a hydraulicbrake system is located?

Answer: b. Unless you determine the source of the fluid escape, youcannot restore the system to its properly closed function.

LESSON 2

Page 43: AT - 4 - Zohowinlearning.zohosites.com › files › PDF › Applied... · The use of materials in this manual does not imply any specific results when WIN materials are used with

Applied Technology • 43

6. The leak has been discovered and repaired in a malfunctioninghydraulic brake system. Before driving the vehicle, what should takeplace?

Answer: c. With the wheels removed, have pressure applied to thebrake pedal while observing the brake shoes and pads toensure their proper activation.

IDEAL

LESSON 2

Page 44: AT - 4 - Zohowinlearning.zohosites.com › files › PDF › Applied... · The use of materials in this manual does not imply any specific results when WIN materials are used with

44 • Applied Technology

#

How did you do? No problem, I’ll bet. Let’s hopeyou didn’t choose answer “e” for question number six!I guess you know why!

Here are some other generalizations about fluid thatyou should review.

Concerning pressure

• The denser the fluid, the greater the pressureit exerts. A good example of this principle issalt water. It is denser than fresh water, andtherefore heavier. It exerts more pressure thanthe same amount of fresh water.

• Fluids seek equilibrium. In other words, theyseek their own level. A fluid will flow froman area of high pressure to low pressure.

• A fluid will not rise higher than its sourcewithout an external force. For example, waterthat is in an underground stream will notrise higher unless there is a pump to force ithigher. The basic well system works on thisprinciple.

LESSON 2

Page 45: AT - 4 - Zohowinlearning.zohosites.com › files › PDF › Applied... · The use of materials in this manual does not imply any specific results when WIN materials are used with

Applied Technology • 45

_

QConcerning evaporation

• The higher a liquid’s temperature, the fasterit will evaporate.

• The lower a liquid’s pressure, the faster it willevaporate.

• The more area of a liquid that is exposed toair, the faster the liquid will evaporate.

• The more circulation of air above a liquid,the faster the liquid will evaporate.

Concerning boiling point

• Increased pressure on a liquid will raise theliquid’s boiling point.

• Decreased pressure on a liquid will lower aliquid’s boiling point.

LESSON 2

Page 46: AT - 4 - Zohowinlearning.zohosites.com › files › PDF › Applied... · The use of materials in this manual does not imply any specific results when WIN materials are used with

46 • Applied Technology

ELECTRICITY

Electricity can be defined as an invisible force thatcan produce heat, light, and motion. It can be explainedin terms of electric charge, voltage, and current. It isthe motion of flow from one atom to another.

Electricity is the continuous flow of electrons, orcurrent, from one atom to another. No electron flowwill occur unless there is a pathway over which theelectrons can move. This flow is similar to a watersystem, where pipes or hoses move water from storagetanks to where it is needed. In electrical wiring, thepathway through which electrical current flows is called

a circuit. A simple circuit consists of a powersource, conductors, load, and a device for

controlling current. Let’s take a look ateach one.

LESSON 3

Page 47: AT - 4 - Zohowinlearning.zohosites.com › files › PDF › Applied... · The use of materials in this manual does not imply any specific results when WIN materials are used with

Applied Technology • 47

• In buildings, the power source could be theelectrical generating stations that pump electricityinto residential and commercial buildings. However,other common sources of electrical power includesmall generators and batteries.

• Conductors, or wiring, provide a path for thecurrent, so it can travel from one point to another.

• A load is a device through which electricity produceswork. For example, a lamp is a load that, whenplugged in and turned on, produces light. Otherexamples of loads include heaters, electric motors,and televisions.

• Switches (on-off switches) control when electricalcurrent flows through circuits. Fuses and circuitbreakers are protective devices that control currentby preventing too much current from flowing inthe circuit, which would damage equipment. Whenan excessive amount of electricity passes throughthem, fuses and circuit breakers “blow” or “trip” tostop the flow of electricity through the circuit.

LESSON 3

Page 48: AT - 4 - Zohowinlearning.zohosites.com › files › PDF › Applied... · The use of materials in this manual does not imply any specific results when WIN materials are used with

48 • Applied Technology

In a circuit, resistance lowers the amount ofelectrical energy available to do work. Both wires andload affect resistance. It might be helpful to think of asimilar situation with a hose that is connected to twosprinklers. As water passes through a hose, turns orkinks in the pathway cause friction (which is resistance)that results in a slower flow. In addition, when some ofthe water is diverted to the first sprinkler (which is aload), less water is available for use in the secondsprinkler.

There are two ways or methods of having currentflow. Direct current flows in one direction. In mostcases, direct current is provided to equipment bybatteries (flashlights and portable radios). Alternatingcurrent flows in one direction, then reverses to the otherdirection. Alternating current is provided to equipmentthrough electrical substations in buildings. In theUnited States, common household current reverses 60times per second. This results in 120v 60 cycle AC.The international measure for cycles is defined in hertz(one hertz = 1 cycle per second).

Measurement of electric current

The rate at which electricity flows is calledamperage. It is measured in amperes. A 100-watt bulbrequires a current of approximately 1 ampere to makeit light up completely. Current flow is measured withan ammeter. Most electrically powered equipmentindicate the amount of current needed to operate itproperly.

LESSON 3

Page 49: AT - 4 - Zohowinlearning.zohosites.com › files › PDF › Applied... · The use of materials in this manual does not imply any specific results when WIN materials are used with

Applied Technology • 49

Measurement of electrical pressure

Pressure is applied to electrons to force them to movethrough a conductor and around a circuit. This pressureis measured in volts. The pressure, or voltage, isavailable in wiring circuits all of the time — whetheror not electrical equipment is being used. Voltage ismeasured with a voltmeter.

Calculation of power

The amount of power derived from an electricaldevice or system is its wattage. In other words, it is theproduct obtained from electrical energy; it is the powerthat we put into use. For example, the electric companysells electrical energy. Electrical energy or power ismeasured in watts and can be calculated as follows:

For direct-current circuits:volts × amperes = watts

For alternating-current circuits:volts × amperes × power factor = watts

NOTE: Power factors range from 0-1. Large equipment(an electric heater) may have a power factor as high as1; small equipment (a small motor) may have a powerfactor as low as .25.

LESSON 3

Thank goodness forcalculators!

Page 50: AT - 4 - Zohowinlearning.zohosites.com › files › PDF › Applied... · The use of materials in this manual does not imply any specific results when WIN materials are used with

50 • Applied Technology

Ohm’s law

Ohm’s law is a simple formula used to describe therelationship between current (flow), voltage (pressure),and resistance of an electrical circuit. Each componentinteracts to affect the operation of a circuit. In otherwords, because voltage pushes current through aresistance, a change in any of the components will resultin a change in the others. The following three equationsare Ohm’s law rearranged to solve for each of thequantities.

Current = Voltage ÷ Resistance I = E/Ramps = volts ÷ ohms An increase in voltage causes an

increase in electrical current flow. Anincrease in circuit resistance causes adecrease in electrical current flow.

Voltage = Current ××××× Resistance E = I × Rvolts = amps × ohms An increase in current causes an

increase in voltage. An increase inresistance causes an increase in voltage.

Resistance = Voltage ÷ Current R = E/Iohms = volts ÷ amps

LESSON 3

Page 51: AT - 4 - Zohowinlearning.zohosites.com › files › PDF › Applied... · The use of materials in this manual does not imply any specific results when WIN materials are used with

Applied Technology • 51

`GENERALIZATIONS THAT CAN BE MADE ABOUT ELECTRICITY:

• The longer the wire, the greater the resistance. The thinner the wire, the greaterthe resistance.

• An increase in temperature of a wire causes an increase in resistance.• An ordinary electrical cord has two wires; one for flow of current from the

power source and the other for the return ground.• The voltage (pressure) and current (flow of electricity) directly affect how much

power is available to do work. Less energy source or lower flow will result inless electrical power being produced.

• A series circuit has only one path for the flow of current. In a series circuit,objects are placed one after another and the current flows through each ofthem in succession. The current is the same throughout, however, and thevoltage is divided among the objects in the circuit.

• In a parallel circuit, there are 2 or more paths, or branches, for the flow ofcurrent. The current will divide and flow through each of the pathssimultaneously. Every branch has the same voltage and — if the appliances areall the same — will have the same amount of current. The total circuit resistanceis less than any one branch.

• When the batteries are connected in a series, the current is the same; the totalvoltage is the sum of each battery. The terminals are connected +, -, +, -, and soon.

• When batteries are connected in parallel, the total current is the sum of thecurrents in each battery; the total voltage is the same as that of one cell. Theterminals are connected +, +, +, and -, -, -.

OK, now that we have covered some of the basicsof electricity, let’s solve a couple of problems.

I will describe a scenario, or problem, and then Iwill ask a few questions related to it. Study the scenarioand answer the questions as best as you can. Theproblems in this level will be a little more difficult thanthe last one, but I don’t believe you will find them toohard to figure out.

LESSON 3

Page 52: AT - 4 - Zohowinlearning.zohosites.com › files › PDF › Applied... · The use of materials in this manual does not imply any specific results when WIN materials are used with

52 • Applied Technology

EXERCISE – THREE-WAY LIGHT SWITCH

Instructions: Using the scenario and diagram, answer the following questions.

Scenario

Upon returning home late one evening, you flip the three-way light switch at the bottom of your stairway. The lightcomes on as it was designed to and you proceed up the stairs.When you reach the top of the stairway and try to turn thelight off by flipping the other three-way switch, the light willnot turn off. You need to figure out what is wrong.

LESSON 3

Three-Way System

Page 53: AT - 4 - Zohowinlearning.zohosites.com › files › PDF › Applied... · The use of materials in this manual does not imply any specific results when WIN materials are used with

Applied Technology • 53

Identify the problem

1. What is the problem?

a. The light does not turn on with either switch.b. The light does not turn off with the switch at the top of the stairs.c. The light does not turn on with the switch at the bottom of the

stairs.d. The light turns on only when in the “down” position.e. The light always stays on.

Define the problem

2. Where is the MOST PROBABLE location of the problem?

a. the light bulbb. the switch at the bottom of the stairsc. the switch at the top of the stairsd. the wiring running between the switchese. a blown fuse

3. The statement that best describes the defective circuit is:

a. The switch at the bottom of the stairs lacks power.b. The switch at the top of the stairs lacks power.c. The light bulb is bad.d. The switch at the top of the stairs is bad.e. The switch at the bottom of the stairs is bad.

LESSON 3

Page 54: AT - 4 - Zohowinlearning.zohosites.com › files › PDF › Applied... · The use of materials in this manual does not imply any specific results when WIN materials are used with

54 • Applied Technology

Explore alternatives

4. When the switch at the bottom of the stairs is turned to the “on”position, you would expect the switch at the top of the stairs tooperate the light:

a. normally in both positions.b. on only in both positions.c. off only.d. on in the “up” position only.e. on in the “down” position only.

Act on a plan

5. To repair the light, one should plan to:

a. replace the light bulb.b. replace the switch at the top of the stairs.c. replace the switch at the bottom of the stairs.d. replace the fuse.e. rewire between the switches.

Look at the result

6. After the switch at the top of the stairs is replaced with a new switch,it is likely that the light will:

a. operate normally.b. remain on all the time.c. remain off all the time.d. operate normally only when the switch at the bottom of the stairs

is on.e. operate normally only when the switch at the bottom of the stairs

is off.

LESSON 3

Page 55: AT - 4 - Zohowinlearning.zohosites.com › files › PDF › Applied... · The use of materials in this manual does not imply any specific results when WIN materials are used with

Applied Technology • 55

ANSWERS TO EXERCISE

1. What is the problem?

Answer: b. The light does not turn off with the switch at the top of thestairs.

2. Where is the MOST PROBABLE location of the problem?

Answer: c. the switch at the top of the stairs

3. The statement that best describes the defective circuit is:

Answer: d. The switch at the top of the stairs is bad.

IDEAL

LESSON 3

Page 56: AT - 4 - Zohowinlearning.zohosites.com › files › PDF › Applied... · The use of materials in this manual does not imply any specific results when WIN materials are used with

56 • Applied Technology

4. When the switch at the bottom of the stairs is turned to the “on”position, you would expect the switch at the top of the stairs tooperate the light:

Answer: c. off only.(Once the light has been turned off, this switch may beused to turn the light on or off.)

5. To repair the light, one should plan to:

Answer: b. replace the switch at the top of the stairs.

6. After the switch at the top of the stairs is replaced with a new switch,it is likely that the light will:

Answer: a. operate normally.

OK, how did you do on those questions? Are youunderstanding the concepts? If not, go back and reviewas much as you need before we move on.

LESSON 3

Page 57: AT - 4 - Zohowinlearning.zohosites.com › files › PDF › Applied... · The use of materials in this manual does not imply any specific results when WIN materials are used with

Applied Technology • 57

This next section will entail some practice usingthe principle of Ohm’s law. Study the diagrams andtext closely. We will go over some vocabularyconcerning circuits before we proceed to the nextexercise.

Series Circuit: A series circuit has only one path forcurrent. For current to flow, a circuit must becontinuous. Refer to Diagram A.

Parallel Circuit: This is a circuit that has more thanone path. Once again, for the current to flow, the pathhas to be continuous. In a parallel circuit, the totalcircuit current is the sum of the currents through theindividual paths. Refer to Diagram B.

LESSON 3

Page 58: AT - 4 - Zohowinlearning.zohosites.com › files › PDF › Applied... · The use of materials in this manual does not imply any specific results when WIN materials are used with

58 • Applied Technology

Resistor: Also known as a load, a resistor is anythingthat slows down the flow of current.

Circuit Breaker: This is a safety device that turns thecircuit off (or breaks the continuous flow of current) ifthe allowable amount of current is exceeded. You maybe familiar with the breaker system in your own home.Whenever any particular circuit overloads, the breakerwill trip and cut off the flow of electricity to that circuit.

Amp: (or ampere) The unit of measurement for electriccurrent is the amp. A label is generally provided onelectrical equipment to indicate the amount of currentneeded.

Let’s consider a couple of scenarios dealing withcircuits.

Study the following diagram. It is a simple parallelcircuit with three lightbulbs attached to copper wires.The power source is a battery.

LESSON 3

Thinking Activity

Point 1

PowerSource

Page 59: AT - 4 - Zohowinlearning.zohosites.com › files › PDF › Applied... · The use of materials in this manual does not imply any specific results when WIN materials are used with

Applied Technology • 59

What would happen if you disconnected one of the bulbs?

That bulb would go off, of course, but since this is a parallel circuit, the otherswould stay on.

What would happen if you disconnected the wire at Point 1?

Since you have broken the continuous flow of power, all three of the bulbs wouldgo off.

Suppose you placed a circuit breaker or fuse between the power source and Point 1.What would happen if too much power was sent into the circuit?

The circuit breaker or fuse would protect the bulbs from blowing by cutting offthe power at the breaker. The entire circuit would be protected.

Suppose you measured the current at the different parallel circuits. Would thevoltage be the same across each circuit?

Yes, the same applied voltage would be measured across each parallel circuit.

LESSON 3

Page 60: AT - 4 - Zohowinlearning.zohosites.com › files › PDF › Applied... · The use of materials in this manual does not imply any specific results when WIN materials are used with

60 • Applied Technology

Thinking Activity

Study the following diagram.

Can you tell if this is a parallel or series circuit?

It is a parallel circuit.

Calculate the total current for the circuit.

The amps are noted on the diagram. Simply add them up (not including the 3amp power source) and you get 1.6 amps.

Calculate the total current when another 10-ohm resistor is added in parallel tothe circuit.

Notice that the 10-ohm resistor at the third bulb is 1 amp. If you add another10-ohm resistor that would make the total 2.6 amps.

What would happen if you add another 10-ohm resistor to the circuit?

If you add yet another 10-ohm resistor which takes one amp, you would have atotal of 3.6 amps. Notice that the power source only provides 3 amps. The circuitbreaker would trip the circuit since this is a .6 amp overload thereby stopping theflow of current.

Let’s do a couple of exercises now.

T

3ASupply

3A

1 2 3

1A.1A.5A

20Ω 100Ω 10Ω

LESSON 3

Page 61: AT - 4 - Zohowinlearning.zohosites.com › files › PDF › Applied... · The use of materials in this manual does not imply any specific results when WIN materials are used with

Applied Technology • 61

EXERCISE – NEW HOME AMPS

Instructions: Read the scenario and answer the following questions using your knowledge ofelectricity.

Scenario

You have moved to a different home. You are experiencingdifficulty with the electrical system. A 20-amp circuit breakerfor the kitchen has been tripping regularly. You determine thatoperating multiple appliances at the same time creates theproblem. You are currently operating the lights, the ceilingfan, the dishwasher, and the range exhaust.

You need to determine which of the following additionalappliances can be used without tripping the breaker.

mixer and coffee maker can opener and mixer can opener and microwave microwave and toaster

LESSON 3

Page 62: AT - 4 - Zohowinlearning.zohosites.com › files › PDF › Applied... · The use of materials in this manual does not imply any specific results when WIN materials are used with

62 • Applied Technology

Identify the problem

1. What is your assignment?

a. to turn off the dishwasherb. to determine why the dishwasher trips the circuit breakerc. to determine which additional electrical appliances can be used

in the circuit without tripping the circuit breakerd. to realize that the temperature outside is 32 degrees Fahrenheite. to turn down the stereo

Define the problem

2. What do you need to consider to solve this problem?

a. the maximum allowable amperage for this parallel circuitb. the number of outlets in your homec. the capacity of the dishwasherd. the number of appliances being usede. the voltage of the circuit

Explore alternatives

3. What is the BEST way to solve this problem?

a. Try all combinations by doing calculations.b. Replace the circuit breaker.c. Never use your dishwasher.d. Call a repair person.e. Try all combinations by using trial and error.

LESSON 3

Page 63: AT - 4 - Zohowinlearning.zohosites.com › files › PDF › Applied... · The use of materials in this manual does not imply any specific results when WIN materials are used with

Applied Technology • 63

Act on a plan

4. What is the correct combination of appliances?

a. mixer and coffee makerb. can opener and mixerc. can opener and microwaved. microwave and toastere. None of the above combinations are correct.

Look at the result

5. Is the new total 20 amps or less?

a. yesb. no

6. What is the best long-term solution?

a. Use a chart to help choose appliance usage.b. Sell your microwave and toaster.c. Add an additional circuit to the kitchen.d. Put in a larger circuit breaker.

LESSON 3

IDEAL

Page 64: AT - 4 - Zohowinlearning.zohosites.com › files › PDF › Applied... · The use of materials in this manual does not imply any specific results when WIN materials are used with

64 • Applied Technology

ANSWERS TO EXERCISE

1. What is your assignment?

Answer: c. to determine which additional electrical appliances can beused in the circuit without tripping the circuit breaker

2. What do you need to consider to solve this problem?

Answer: a. the maximum allowable amperage for this parallel circuit

3. What is the BEST way to solve this problem?

Answer: a. Try all combinations by doing calculations.

4. What is the correct combination of appliances?

Answer: b. can opener and mixer

5. Is the new total less than or equal to 20 amps or less?

Answer: a. yes

6. What is the best long-term solution?

Answer: c. Add an additional circuit to the kitchen.Putting in a larger circuit breaker would likely create a firehazard as the kitchen wiring may not handle the largercurrent.

LESSON 3

Page 65: AT - 4 - Zohowinlearning.zohosites.com › files › PDF › Applied... · The use of materials in this manual does not imply any specific results when WIN materials are used with

Applied Technology • 65

EXERCISE – PAN CONVEYOR MOTOR

Instructions: After reading the following scenario, answer the questions.

Scenario

An operator is attempting to start up Production Line 7. Heclicks on the “start” button for the pan conveyor and nothinghappens. He notices that there is a message in the alarmwindow that reads, “pan conveyor motor fault.” What shouldthe operator do to investigate and resolve the problem?

___________________________________________________________________

NOTE: There is not a diagram of the pan conveyor system that is involved in this problem.You are encouraged to draw a diagram that represents such a system. (In a pan conveyorsystem, a motor causes the conveyor to vibrate. The conveyor vibrates to move rawmaterials from one point to another in the production process.)___________________________________________________________________

Identify the problem

1. What piece of equipment is the operator having problems with?

a. the pan conveyorb. a motor overload for the pan conveyor motorc. the computer and the PLCd. the Acrison® feederse. none of the above

LESSON 3

Page 66: AT - 4 - Zohowinlearning.zohosites.com › files › PDF › Applied... · The use of materials in this manual does not imply any specific results when WIN materials are used with

66 • Applied Technology

Define the problem

2. The statement that best describes the operator’s problem is:

a. The pan conveyor will not start.b. None of the feeders will run.c. The extruder will not start.d. The “start” button malfunctioned.e. None of the above occurred.

Explore alternatives

3. What is the most likely cause of the problem?

a. a burned-up drive motorb. a tripped motor overloadc. too much material on the pan conveyord. improperly wired motor circuite. none of the above

Act on the plan

4. The operator should:

a. call his or her supervisor for advice.b. call maintenance to check out the problem.c. check the motor starter to see if the overload is tripped; if so,

reset it.d. acknowledge the alarm and try to start the conveyor again.e. do none of the above.

LESSON 3

Page 67: AT - 4 - Zohowinlearning.zohosites.com › files › PDF › Applied... · The use of materials in this manual does not imply any specific results when WIN materials are used with

Applied Technology • 67

Look at the result

5. Once the problem has been corrected, how can it be avoided in thefuture?

a. Notify maintenance of what happened and have them check forconditions that would have caused the overload.

b. Have maintenance wire the motor correctly.c. Do both a and b.d. Do none of the above.

IDEAL

LESSON 3

Page 68: AT - 4 - Zohowinlearning.zohosites.com › files › PDF › Applied... · The use of materials in this manual does not imply any specific results when WIN materials are used with

68 • Applied Technology

ANSWERS TO EXERCISE

1. What piece of equipment is the operator having problems with?

Answer: a. the pan conveyor

2. The statement that best describes the operator’s problem is:

Answer: a. The pan conveyor will not start.

3. What is the most likely cause of the problem?

Answer: b. a tripped motor overload

4. The operator should:

Answer: c. Check the motor starter to see if the overload is tripped; ifso, reset it.

5. Once the problem has been corrected, how can it be avoided in thefuture?

Answer: a. Notify maintenance of what happened and have themcheck for conditions that would have caused the overload.

LESSON 3

Page 69: AT - 4 - Zohowinlearning.zohosites.com › files › PDF › Applied... · The use of materials in this manual does not imply any specific results when WIN materials are used with

Applied Technology • 69

MECHANICS

A machine is simply something that does work.Work is done when a force causes an object to move.Man invented machines thousands of years ago whenhe realized the limitation of the human body as itapplied to basic survival.

For example, in the Stone Age, man realized thathe did not have the physical ability to obtain food withconsistent success. He needed something to help himsurvive. He learned to use a club, a rock, and later,invented the spear to help him hunt for food. Trapswere devised to catch certain animals. All of these thingsdid work, and they were machines.

Later, more complex machines were invented tomake his work easier and faster. He discovered that bytaking a long pole and placing it on a raised object (thefulcrum) he could lift boulders many times larger thanones he could lift with his own physical strength. Hehad discovered the lever! The lever is a simple machine,but wow, what a difference it makes!

All simple machines, i.e., gears, pulleys, inclinedplanes, levers, wheel and axle, are used to make up morecomplex machines. These type of machines are calledcompound machines. When you think of thecomplexity of an average automobile engine, you canimagine that it is a combination of simple machinesput together to form the completed engine!

LESSON 4

Page 70: AT - 4 - Zohowinlearning.zohosites.com › files › PDF › Applied... · The use of materials in this manual does not imply any specific results when WIN materials are used with

70 • Applied Technology

If you have ever toured or worked in amanufacturing plant, you may have been amazed atthe machinery and how complex it was. Thousands ofsimple machines work together to produce some object.Without such machinery, almost nothing we use intoday’s modern world would be possible. Because ofman’s ingenuity, perseverance, experimentation, andeffort, we have the complicated machines that we usein every day life. We are so accustomed to having theseconveniences, we usually aren’t even aware of thecomplexity of them!

In this lesson we will examine the principle of torque,which involves center of mass, balance, and rotationalmovement.

The principle of torque has a wide variety of usesin the workplace. Some of these include wheel rotation,machine operations, sawing, and other technicalsystems that rotate.

Torque is an important concept when studyingbalance. There is a moment arm and a hanging weight.When the hanging weight is farther from the pivotpoint, the torque is greater. The pivot point is the centerof the system’s mass when the system is in balance.

LESSON 4

Page 71: AT - 4 - Zohowinlearning.zohosites.com › files › PDF › Applied... · The use of materials in this manual does not imply any specific results when WIN materials are used with

Applied Technology • 71

The torque on each side of the pivot point must beequal. Consider the teeter-totter for an example of thisconcept. A heavy weight needs to be moved closer tothe center (or pivot point) to achieve less torque whilea light weight should be moved out from the pivotpoint to achieve more torque. In order to make theteeter-totter balance on the pivot point, (or center) theweight will be moved accordingly until a balance isachieved.

Another good example of balance and torque is thespin cycle of your washing machine. If the clothes inthe washer are out of balance around the center of themachine, you hear a terrible banging noise that soundslike the machine is about to shake itself to pieces! Youmust stop it and rearrange the clothes so that thebalance is restored. The same principle applies to asimple machine such as the lever. You know that thelonger the rod that is placed on the fulcrum, the easierthe load will be to lift, or the torque is greater.

Another example of a common object is the fan. Ifthe blades are out of balance because of different sizesor weights, the fan will not rotate properly.

Let’s examine the following diagrams to visualizesome of these examples.

LESSON 4

Pop Quiz: The rate atwhich electricity flows ismeasured in ________?

Page 72: AT - 4 - Zohowinlearning.zohosites.com › files › PDF › Applied... · The use of materials in this manual does not imply any specific results when WIN materials are used with

72 • Applied Technology

Notice the placement of weight in respect tothe pivot point. This is necessary to achievebalance. The larger weight is closer to thepivot point (less torque), while the smallerweight is moved farther from the pivot point(more torque).

Will this fan rotate properly?(No, the blades are different sizes and therefore different weights.)

Now that we have examined the principle of torque,let’s solve some problems.

LESSON 4

Page 73: AT - 4 - Zohowinlearning.zohosites.com › files › PDF › Applied... · The use of materials in this manual does not imply any specific results when WIN materials are used with

Applied Technology • 73

EXERCISE – LAWN MOWER

Instructions: Answer the following questions based on the following scenario and diagram.

Scenario

You own a lawn mowing service and you use a 13-hp ridinglawn mower with twin cutting blades mounted under a 42"deck. The engine powers the blades by use of a belt-and-pulleysystem, which includes a clutch that can be engaged whendesired. You just finished mowing a lawn with some roughterrain. You transport the riding lawn mower to the next job.Upon arriving at the next site, you start the engine. The motorstarts easily and runs smoothly. Upon engaging the cuttingblades, you notice some unusual and excessive vibration butnot any unusual noises. The vibration increases in intensityas you increase the engine speed. You need to determine thecause of this unwanted vibration. Refer to the diagram toanswer the following questions.

LESSON 4

Page 74: AT - 4 - Zohowinlearning.zohosites.com › files › PDF › Applied... · The use of materials in this manual does not imply any specific results when WIN materials are used with

74 • Applied Technology

Identify the problem

1. What is your assignment?

a. Continue to mow the lawn.b. Accelerate the engine.c. Decelerate the engine.d. Find the cause of the vibration.e. Disengage the blade and continue.

Define the problem

2. When did the vibration start?

a. when you started the engineb. when you sat down on the lawn mower’s seatc. when you transported the lawn mower between mowing sitesd. when you increased the engine speede. when you engaged the mower blades

Explore alternatives

3. What is the most likely cause of the vibrations?

a. bent engine crankshaftb. cutting deckc. loose tire mountd. low engine oile. improperly engaged clutch

LESSON 4

Page 75: AT - 4 - Zohowinlearning.zohosites.com › files › PDF › Applied... · The use of materials in this manual does not imply any specific results when WIN materials are used with

Applied Technology • 75

Act on a plan

4. What should be done FIRST to eliminate the vibration?

a. Adjust the blade elevation.b. Balance the blade.c. Adjust the belt tension.d. Visually inspect the underside of the mower deck for irregularities.e. Sharpen the blades.

Look at the result

5. You inspected the underside of the deck and found a bent cuttingblade. What should you do next?

a. Replace the blade.b. Sharpen the blade.c. Lower the blade so that it doesn’t hit the deck housing.d. Lubricate all of the bearings.e. Adjust the tension in the drive belt.

LESSON 4

IDEAL

Page 76: AT - 4 - Zohowinlearning.zohosites.com › files › PDF › Applied... · The use of materials in this manual does not imply any specific results when WIN materials are used with

76 • Applied Technology

ANSWERS TO EXERCISE

1. What is your assignment?

Answer: d. Find the cause of the vibration.

2. When did the vibration start?

Answer: e. when you engaged the mower blades

3. What is the most likely cause of the vibrations?

Answer: b. cutting deck

4. What should be done FIRST to eliminate the vibration?

Answer: d. Visually inspect the underside of the mower deck forirregularities.

5. You inspected the underside of the deck and found a bent cuttingblade. What should you do next?

Answer: a. Replace the blade.

LESSON 4

Page 77: AT - 4 - Zohowinlearning.zohosites.com › files › PDF › Applied... · The use of materials in this manual does not imply any specific results when WIN materials are used with

Applied Technology • 77

Well, that’s it for this level of Applied Technology.How did you do? Are you ready for the Posttest? Ifnot, go back and review. Take the test when you feelready. Remember, don’t peek at the answers becausethat won’t give you an accurate assessment of yourprogress. OK, go for it and good luck!

LESSON 5

No peeking allowed!

Page 78: AT - 4 - Zohowinlearning.zohosites.com › files › PDF › Applied... · The use of materials in this manual does not imply any specific results when WIN materials are used with

78 • Applied Technology

EXERCISE – POSTTEST

Instructions: Answer the following questions about the principles of applied technology.

1. What is conduction?

____________________________________________________________

____________________________________________________________

2. What is one of the most common and best conductors of heat orelectricity?

____________________________________________________________

3. ______________ takes in heat while ______________ turns it back.

4. ________ _____________ is what happens when heat is passedthrough a conductive material into another object.

5. Give an example where friction would be a constructive force.

____________________________________________________________

____________________________________________________________

6. Give an example where friction would be a destructive force.

____________________________________________________________

____________________________________________________________

POSTTEST

Page 79: AT - 4 - Zohowinlearning.zohosites.com › files › PDF › Applied... · The use of materials in this manual does not imply any specific results when WIN materials are used with

Applied Technology • 79

7. While running a machine, you notice that the bearing assemblyhousing is beginning to smoke. What do you do?

____________________________________________________________

____________________________________________________________

8. As resistance increases, fluid flow _________________.

9. Cite an example illustrating the principle described in question 8.

____________________________________________________________

____________________________________________________________

10. Cite an example where this principle can be used to achieve theopposite result.

____________________________________________________________

____________________________________________________________

11. You are filling your aquarium with water when you notice that thewater flow has decreased dramatically. What could you check tofind the problem?

____________________________________________________________

____________________________________________________________

POSTTEST

Page 80: AT - 4 - Zohowinlearning.zohosites.com › files › PDF › Applied... · The use of materials in this manual does not imply any specific results when WIN materials are used with

80 • Applied Technology

12. _____________ ________ states that pressure applied to anenclosed fluid is transmitted equally in all directions to everyportion of the fluid and to the walls of the container.

13. The higher the fluid’s temperature, the faster it will _____________.

14. A _____________ circuit has only one path.

15. A ______________ circuit has more than one path.

16. What is the purpose of a circuit breaker?

____________________________________________________________

____________________________________________________________

17. What else can be used for the same purpose?

____________________________________________________________

18. The measurement of the rate at which the electric current flows isthe ______________________.

19. ______________ is anything that slows down the flow of current.

20. In a parallel circuit, the total circuit current is the ______ of thecurrents through the individual paths.

POSTTEST

Page 81: AT - 4 - Zohowinlearning.zohosites.com › files › PDF › Applied... · The use of materials in this manual does not imply any specific results when WIN materials are used with

Applied Technology • 81

21. _______________ involves center of mass, balance, and rotationalmovement.

22. The torque on each side of the pivot point must be _____________to achieve proper balance.

23. If a heavy weight is moved closer to the pivot point, the torque willbe (more, less). (Circle the correct word.)

24. You have two weights placed on a board; one is 5 lb and the other is2 lb. Which one should be farther from the pivot point to achieveperfect balance?

____________________________________________________________

25. You are dismantling a bookcase when you encounter a slotted screwthat is especially hard to remove. What should you do?

____________________________________________________________

____________________________________________________________

26. Fan blades must be of equal weight and distance around the centermass or the fan will ___________.

POSTTEST

Page 82: AT - 4 - Zohowinlearning.zohosites.com › files › PDF › Applied... · The use of materials in this manual does not imply any specific results when WIN materials are used with

82 • Applied Technology

27. You are attempting to remove a heavy stone with a lever, but youfind that even if you put all of your weight on the lever pole, you canonly lift it a few inches. What should you do?

a. blast it out with dynamiteb. use a jackc. get a longer lever

POSTTEST

Page 83: AT - 4 - Zohowinlearning.zohosites.com › files › PDF › Applied... · The use of materials in this manual does not imply any specific results when WIN materials are used with

Applied Technology • 83

ANSWERS TO EXERCISE

1. What is conduction?

Answer: Conduction is the passage of energy from particle to particle.

2. What is one of the most common and best conductors of heat orelectricity?

Answer: metal

3. _______________ takes in heat while ____________ turns it back.

Answer: Absorption, reflection

4. __________ _____________ is what happens when heat is passedthrough a conductive material into another object.

Answer: Heat transfer

5. Give an example where friction would be a constructive force.

Answer: Answers may vary.• trying to start a fire• automobile brakes• clutches• parachutes

POSTTEST

Page 84: AT - 4 - Zohowinlearning.zohosites.com › files › PDF › Applied... · The use of materials in this manual does not imply any specific results when WIN materials are used with

84 • Applied Technology

6. Give an example where friction would be a destructive force.

Answer: Answers may vary.• With no oil in a car engine, the engine could be damaged.• Friction causes the tread on tires to wear down.• Without lubricant, bearings would not be effective.

7. While running a machine, you notice that the bearing assemblyhousing is beginning to smoke. What do you do?

Answer: • Stop the machine.• Let it cool off.• Check for oil in the bearing case housing.

8. As resistance increases, fluid flow ______________.

Answer: decreases

9. Cite an example illustrating the principle described in question 8.

Answer: Answers may vary.running more than one faucet at a time

10. Cite an example where this principle can be used to achieve theopposite result.

Answer: Answers may vary.a water saver nozzle, that would force the water throughsmaller holes (In this case, resistance at the end of the flowwould be increased.)

POSTTEST

Page 85: AT - 4 - Zohowinlearning.zohosites.com › files › PDF › Applied... · The use of materials in this manual does not imply any specific results when WIN materials are used with

Applied Technology • 85

11. You are filling your aquarium with water when you notice that thewater flow has decreased dramatically. What could you check tofind the problem?

Answer: • Check to see if another water outlet (faucet, washer, etc.)is running.

• Check for a kink in the hose.

12. ____________ __________ states that pressure applied to anenclosed fluid is transmitted equally in all directions to everyportion of the fluid and to the walls of the container.

Answer: Pascal’s law

13. The higher the fluid’s temperature, the faster it will _________.

Answer: evaporate

14. A _____________ circuit has only one path.

Answer: series

15. A _______________ circuit has more than one path.

Answer: parallel

16. What is the purpose of a circuit breaker?

Answer: to protect from overloaded circuits

POSTTEST

Page 86: AT - 4 - Zohowinlearning.zohosites.com › files › PDF › Applied... · The use of materials in this manual does not imply any specific results when WIN materials are used with

86 • Applied Technology

17. What else can be used for the same purpose?

Answer: a fuse

18. The measurement of the rate at which the electric current flows isthe _____________________.

Answer: amp or ampere

19. ______________ is anything that slows down the flow of current.

Answer: Resistance

20. In a parallel circuit, the total circuit current is the ______ of thecurrents through the individual paths.

Answer: sum

21. _______________ involves center of mass, balance, and rotationalmovement.

Answer: Torque

22. The torque on each side of the pivot point must be ______ to achieveproper balance.

Answer: equal

POSTTEST

Page 87: AT - 4 - Zohowinlearning.zohosites.com › files › PDF › Applied... · The use of materials in this manual does not imply any specific results when WIN materials are used with

Applied Technology • 87

23. If a heavy weight is moved closer to the pivot point, the torque willbe _____.

Answer: less

24. You have two weights placed on a board; one is 5 lb and the other is2 lb. Which one should be farther from the pivot point to achieveperfect balance?

Answer: the smaller 2 lb weight

25. You are dismantling a bookcase when you encounter a slotted screwthat is especially hard to remove. What should you do?

Answer: Insure the screwdriver being used has a large enough bladeto totally fill the screw slot. This provides maximum torque.

26. Fan blades must be of equal weight and distance around the centermass or the fan will ___________.

Answer: wobble, vibrate, or rotate unbalanced

27. You are attempting to remove a heavy stone with a lever. You findthat even if you put all of your weight on the lever pole, you can onlylift it a few inches. What should you do?

Answer: c. get a longer lever

POSTTEST

Page 88: AT - 4 - Zohowinlearning.zohosites.com › files › PDF › Applied... · The use of materials in this manual does not imply any specific results when WIN materials are used with

88 • Applied Technology

Calculate your score counting the number of questions you answered correctly. If aproblem asked you to list several items or steps and you missed one or more, count thequestion as answered incorrectly. Divide the number of your correct answers by 27.Change the decimal answer to a percentage by moving the decimal two places to theright.

CALCULATING YOUR SCORE

Page 89: AT - 4 - Zohowinlearning.zohosites.com › files › PDF › Applied... · The use of materials in this manual does not imply any specific results when WIN materials are used with

Applied Technology • 89

How well did you do on the Posttest? If you scored 93% or higher, you have a reasonable chance to pass Level 4 of the Applied Technology assessment. Remember to use the IDEAL model when solving problems.

I = Identify the problemD = Define and represent the problem

boundariesE = Explore alternative approachesA = Act on a planL = Look at the result

Now, don’t be discouraged if you scored below 93%.Practice the exercises in this course – you can do it.Your enhanced work skills will pay off in the long run.Don’t forget to practice your problem-solving skills.

SUMMARY

You are making progress!

Page 90: AT - 4 - Zohowinlearning.zohosites.com › files › PDF › Applied... · The use of materials in this manual does not imply any specific results when WIN materials are used with

90 • Applied Technology

EDWIN’S TEST–TAKING TIPS

Preparing for the test . . .

Complete appropriate levels of the WIN Instruction Solution self-study courses. Practice the exercises until you begin to feel comfortable solving problems.

Get a good night’s rest the night before the test and eat a good breakfast on test day. Your body (specifically your mind) works better when you take good care of it.

You should take the following items with you when you take the Applied Technology assessment: (1) pencils; pens are not allowed to be used on the test; it is a good idea to have more than one pencil since the test is timed and you do not want to waste time sharpening a broken pencil lead; and (2) your calculator; be sure your batteries are strong if you do not have a solar-powered calculator and that your calculator is working properly. Pencils will be provided for those who need them.

Allow adequate time to arrive at the test site. Being in a rush or arriving late will likely upset your concentration when you actually take the test.

About the test . . .

The test is comprised of approximately 32 multiple-choice questions. The questionscover four areas: thermodynamics, fluid dynamics, electricity, and mechanics. Somequestions will be presented as single questions while others may be in groups of twoand refer to specific figures or scenarios. You will not be penalized for wrong answers,so it is better to guess than leave blanks. You will have 45 minutes to complete thetest.

You will not be allowed to use scratch paper, but there is room in your assessmentbooklet to make calculations in solving.

REFERENCE

Page 91: AT - 4 - Zohowinlearning.zohosites.com › files › PDF › Applied... · The use of materials in this manual does not imply any specific results when WIN materials are used with

Applied Technology • 91

During the test . . .

Listen to instructions carefully and read the test booklet directions. Do not hesitateto ask the administrator questions if you do not understand what to do.

Pace yourself since this is a timed test. The administrator will let you know when youhave 5 minutes left and again when you have 1 minute remaining. Work as quickly aspossible, but be especially careful as you enter numbers into your calculator.

If a problem seems too difficult when you read it, skip over it (temporarily) and moveon to an easier problem. Be sure to put your answers in the right place. Sometimesskipping problems can cause you to get on the wrong line, so be careful. You mightwant to make a mark in the margin of the test, so that you will remember to go backto any skipped problems.

Since this is a multiple-choice test, you have an advantage answering problems thatare giving you trouble. Try to eliminate any unreasonable answers and make aneducated guess from the answers you have left.

If the administrator indicates you have one minute remaining and you have someunanswered questions, be sure to fill in an answer for every problem. Your guess isbetter than no answer at all!

If you answer all of the test questions before time is called, use the extra time to checkyour answers. It is easy to hit the wrong key on a calculator or place an answer on thewrong line when you are nervous. Look to see that you have not accidentally omittedany answers.

REFERENCE

Page 92: AT - 4 - Zohowinlearning.zohosites.com › files › PDF › Applied... · The use of materials in this manual does not imply any specific results when WIN materials are used with

92 • Applied Technology

Dealing with test anxiety . . .

Being prepared is one of the best ways to reduce test anxiety. Remember to use thefive steps that we used in solving problems: identify the problem, define the problem,explore alternatives, act on a plan, and look at the result. Identifying several ways tosolve problems and following a systematic process should increase your confidenceand reduce anxiety.

Do not think negatively about the test. The story about the “little engine that could”is true. You must, “think you can, think you can, think you can.” If you prepareyourself by studying problem-solving strategies, there is no reason why you cannot besuccessful.

Do not expect yourself to know how to solve every problem. Do not expect to knowimmediately how to work the problems when you read them. Everyone has to readand reread problems when they are solving problems. So, don’t get discouraged; bepersistent.

Prior to the test, close your eyes, take several deep breaths, and think of a relaxingplace or a favorite activity. Visualize this setting for a minute or two before the test isadministered.

During the test if you find yourself tense and unable to think, try the followingrelaxation technique:

1. Put your feet on the floor.2. Grab under your chair with your hands. (Hope there are no surprises!)3. Push down with your feet and up on your chair at the same time - hold for 5

seconds.4. Relax 5 seconds (especially try to relax your neck and shoulders).5. Repeat a couple of times as needed, but do not spend the entire 45 minutes of

test trying to relax!

Studying with a partner is another way to overcome test anxiety. Encouragementfrom each other helps to increase your confidence.

REFERENCE

Page 93: AT - 4 - Zohowinlearning.zohosites.com › files › PDF › Applied... · The use of materials in this manual does not imply any specific results when WIN materials are used with

Applied Technology • 93

BASIC SCIENTIFIC PRINCIPLES

Applied Technology focuses on:• Principles related to power sources - for mechanical, electrical, thermal, and fluid

systems• Principles related to flow - for mechanical, electrical, thermal, and fluid systems• Principles related to pressure - for mechanical, electrical, thermal, and fluid systems• Principles related to resistance - for mechanical, electrical, thermal, and fluid systems

The basic scientific principles involved with energy sources, flow, pressure, and resistanceappear below:

Bernoulli’s principle: The faster the flow of air or fluid, the lower the pressure.

Boyle’s law: For a certain amount of gas, at a constant temperature, as the pressure (P)increases, the volume (V) of the gas decreases so that P times V is constant (k). (PV=k).

Charles’ law: For a certain amount of gas, at a constant pressure, as the absolutetemperature of the gas increases, the volume of the gas also increases. Mathematicallythis is: Volume (V) divided by temperature (T) equals a constant (k). V/T=k Thetemperature must be on an absolute scale that is in reference to absolute zero.

Hooke’s law: The greater the force exerted on an object, the more it will be moved. Forexample, the heavier the weight hanging from a spring, the more the spring will bestretched.

Newton’s laws of motion• An object will remain at rest or in uniform motion unless acted upon by an outside

force.• When a force acts upon an object, it changes the momentum of that object, and

this change is proportional to the applied force and to the time that it acts uponthe object.

• Every action (force) is followed by an equal and opposite reaction (force).

REFERENCE

Page 94: AT - 4 - Zohowinlearning.zohosites.com › files › PDF › Applied... · The use of materials in this manual does not imply any specific results when WIN materials are used with

94 • Applied Technology

Laws of Thermodynamics• Energy cannot be created or destroyed.• Heat energy always flows spontaneously from hot to cold.

Ohm’s law: Current is directly proportional to the voltage and inversely proportional tothe resistance.

Pascal’s law: Pressure added to a confined fluid at any point instantly appears equally atall other points and is always at right angles to the containing surfaces.

REFERENCE

EdWIN’s LAW

Mind over matter can

make people happy!

Page 95: AT - 4 - Zohowinlearning.zohosites.com › files › PDF › Applied... · The use of materials in this manual does not imply any specific results when WIN materials are used with

Applied Technology • 95

REFERENCE

Generalizations that can be made about mechanics:

A machine is something that does work.

Work is done when a force causes an object to move.

Simple machines (gears, pulleys, inclined planes, levers, wheel and axle), which aredescribed below, make up compound (or complex) machines.

Compound machines include a bicycle, a rod and reel, a typewriter, a can opener, scissors,a hand drill, a car, a weight machine, and a treadmill.

Gears• The force that is applied to a driver gear is transferred to a driven gear.• When two gears of different sizes are meshed together, the smaller gear turns

faster (more rotations per minute) than the larger gear.• Gears that are meshed together move in opposite directions.• The direction and speed of the driver gear determines the speed and direction of

gears that are meshed with it.• Common applications of gears include bicycle sprocket chains, speedometers,

clocks, electric mixers, lawn sprinklers, and egg beaters.

Pulleys• A pulley is a wheel with a rope, belt, or chain around it.• Pulleys change the direction of movement and make work easier.• Fixed pulleys change the direction that something is moved; they do not make

work easier.• Movable pulleys change the direction that something is moved and make work

easier.• The more pulleys in the system, the easier it is to do work (pull or lift an object).• The more pulleys involved in a system, the greater distance must be pulled, but

the easier it is to do work.• The thinner the windlass (winch), the easier it is to turn.• In two different sets of pulleys, if the wheels are connected by a shaft and the two

wheels on one pulley are the same size as the two wheels on the other pulley, theywill both turn at the same speed.

• Common pulley applications include crankshafts, sailboats, boat lifts, windowblinds, cranes, elevators, and escalators.

Page 96: AT - 4 - Zohowinlearning.zohosites.com › files › PDF › Applied... · The use of materials in this manual does not imply any specific results when WIN materials are used with

96 • Applied Technology

REFERENCE

Inclined Planes• An inclined plane is a slanted surface that is used to raise or lower heavy objects

from one position to another.• Inclined planes help reduce the amount of force needed to do a given amount of

work, but require greater distance.• The steeper the plane, the more difficult the work.• Wedges are two back-to-back inclined planes.• Common applications of inclined planes include a screw, a bolt, a drill bit, a

clamp, a car jack, and a screw-on bottle top.

Levers• A lever is a bar or rod that is free to move or turn on a fulcrum.• A lever multiplies force, but some distance must be given up.• The shorter the effort arm, the less force is attained and the greater distance is

attained.• The longer the effort arm, the more force is attained and the less distance is attained.• Examples of levers include scissors, a broom, a claw hammer, a nutcracker, a mop,

tongs, a crowbar, a can opener, tweezers, a baseball bat, boat oars, and a car jackhandle.

Wheel and Axle• A wheel and axle is like a spinning lever (an ice cream machine crank).• The center of the axle is the fulcrum.• The wheel is larger than the axle; for one rotation, a point on the edge of the

wheel travels a greater distance than a point on the axle. While the work doneby the axle and the wheel are the same, the greater distance traveled of thepoint on the edge of the wheel yields a smaller force at the edge of the wheelversus the edge of the axle.

• Common wheel and axle applications include a screwdriver, roller skates,a water-faucet handle, a bicycle pedal, a can opener, and a car steering wheel.

Page 97: AT - 4 - Zohowinlearning.zohosites.com › files › PDF › Applied... · The use of materials in this manual does not imply any specific results when WIN materials are used with

Applied Technology • 97

REFERENCE

Overview of Electricity

Electricity is the continuous flow of electrons, or current, from one atom to another. Noelectron flow will occur unless there is a pathway over which the electrons can move.This flow is similar to a water system, where pipes or hoses move water from storagetanks to where it is needed. In electrical wiring, the pathway through which electricalcurrent flows is called a circuit. A simple circuit consists of a power source, conductors,load, and a device for controlling current. Each is described below.

In buildings, the power source could be the electrical generating stations that pumpelectricity into residential and commercial buildings. However, other common sourcesof electrical power include small generators and batteries.

Conductors, or wiring, provide a path for the current so that it can travel from one pointto another.

A load is a device through which electricity produces work. For example, a lamp is a loadthat, when plugged in and turned on, produces light. Other examples of loads includeheaters, electric motors, and televisions.

Switches (on-off switches) control when electrical current flows through circuits. Fusesand circuit breakers are protective devices that control current by preventing too muchcurrent from flowing in the circuit, which would damage equipment. When an excessiveamount of electricity passes through them, fuses and circuit breakers “blow” or “trip” tostop the flow of electricity through the circuit.

In a circuit, resistance lowers the amount of electrical energy available to do work. Bothwires and load affect resistance. It might be helpful to think of a similar situation with ahose that is connected to two sprinklers. As water passes through a hose, turns or kinksin the pathway cause friction (which is resistance) that results in a slower flow. In addition,when some of the water is diverted to the first sprinkler (which is a load), less water isavailable for use in the second sprinkler.

Page 98: AT - 4 - Zohowinlearning.zohosites.com › files › PDF › Applied... · The use of materials in this manual does not imply any specific results when WIN materials are used with

98 • Applied Technology

REFERENCE

There are two ways or methods of having current flow. Direct current flows in onedirection. In most cases, direct current is provided to equipment by batteries (flashlightsand portable radios). Alternating current flows in one direction, then reverses to theother direction. Alternating current is provided to equipment through electrical substationsin buildings. In the United States, common household current reverses itself 60 timesper second. This results in 120v 60 cycle AC. The international reference for cycles isdefined in hertz (one hertz = 1 cycle per second).

Measurement of Electric Current

The rate at which electricity flows is called amperage. It is measured in amperes. A 100-watt bulb requires a current of approximately 1 ampere to make it light up completely.Current flow is measured with an ammeter. Most electrically powered equipment indicatethe amount of current needed to operate it properly.

Measurement of Electrical Pressure

Pressure is applied to electrons to force them to move through a conductor and around acircuit. This pressure is measured in volts. The pressure, or voltage, is available in wiringcircuits all of the time - whether or not electrical equipment is being used. Voltage ismeasured with a voltmeter.

Calculation of Power

The amount of power derived from an electrical device or system is its wattage. In otherwords, it is the product obtained from electrical energy; it is the power that we put intouse. For example, the electric company sells electrical energy. Electrical energy or poweris measured in watts and can be calculated as follows:

For direct-current circuits: volts ××××× amperes = watts

For alternating-current circuits: volts ××××× amperes ××××× power factor = watts

NOTE: Power factors range from 0-1. Large equipment (an electric heater) may have apower factor as high as 1; small equipment (a small motor) may have a power factor aslow as .25.

Page 99: AT - 4 - Zohowinlearning.zohosites.com › files › PDF › Applied... · The use of materials in this manual does not imply any specific results when WIN materials are used with

Applied Technology • 99

REFERENCE

Ohm’s law

Ohm’s law is a simple formula used to describe the relationship between current (flow),voltage (pressure), and resistance of an electrical circuit. Each component interacts toaffect the operation of a circuit. In other words, because voltage pushes current througha resistance, a change in any of the components will result in a change in the others. Thefollowing three equations are Ohm’s law rearranged to solve for each of the quantities:

Current = Voltage ÷ Resistance I = E/Ramps = volts ÷ ohms An increase in voltage causes an increase in

electrical current flow. An increase in circuitresistance causes a decrease in electrical currentflow.

Voltage = Current ××××× Resistance E = I ××××× Rvolts = amps × ohms An increase in current causes an increase in

voltage. An increase in resistance causes anincrease in voltage.

Resistance = Voltage ÷ Current R = E/Iohms = volts ÷ amps

Page 100: AT - 4 - Zohowinlearning.zohosites.com › files › PDF › Applied... · The use of materials in this manual does not imply any specific results when WIN materials are used with

100 • Applied Technology

REFERENCE

Generalizations that can be made about electricity:

• The longer the wire, the greater the resistance; the thinner the wire, the greaterthe resistance.

• An increase in temperature of a wire causes an increase in resistance.

• An ordinary electrical cord has two wires; one for the flow of current from thepower source and the other for the return or ground.

• The voltage (pressure) and current (flow of electricity) directly affect how muchpower is available to do work. Less energy source or lower flow will result in lesselectrical power being produced.

• A series circuit has only one path for the flow of current. In a series circuit, objectsare placed one after another and the current flows through each of them insuccession. The current is the same throughout, however, and the voltage is dividedamong the objects in the circuit.

• In a parallel circuit, there are 2 or more paths, or branches, for the flow of current.The current will divide and flow through each of the paths simultaneously. Everybranch has the same voltage and - if the appliances are all the same - will have thesame amount of current. The total circuit resistance is less than any one branch.

• When batteries are connected in a series, the current is the same; the total voltageis the sum of the voltage of each battery. The terminals are connected +, -, +,-, andso on.

• When batteries are connected in parallel, the total current is the sum of the currentsin each battery; the total voltage is the same as that of one cell. The terminals areconnected +, +, +, and -, -, -.

Page 101: AT - 4 - Zohowinlearning.zohosites.com › files › PDF › Applied... · The use of materials in this manual does not imply any specific results when WIN materials are used with

Applied Technology • 101

REFERENCE

Generalizations that can be made about heat:

• Heat travels through conductors (metal) better than through insulators (wood).

• Dark-colored surfaces absorb more heat than light-colored surfaces.

• Rough or dull surfaces absorb more heat than smooth or shiny surfaces.

• When friction causes heat, the object that is in constant contact gets hotter thanthe movable object. (For example, the wood being cut gets hotter than the sawblade; car brake shoes get hotter than the wheel.)

Generalizations that can be made about fluids:

Pressure• The amount of pressure exerted by a fluid depends upon the height and the density

of that fluid and is independent of the shape of the container that is holding thefluid.

• The deeper the fluid, the greater the pressure it exerts.

• The denser the fluid, the greater the pressure it exerts (salt water is denser thanfresh water).

• Fluids seek equilibrium - they seek their own level; a fluid will flow from a place ofhigh pressure to a place of low pressure.

• A fluid can never rise higher than its source without an external force (a pump).

Page 102: AT - 4 - Zohowinlearning.zohosites.com › files › PDF › Applied... · The use of materials in this manual does not imply any specific results when WIN materials are used with

102 • Applied Technology

REFERENCE

Evaporation• The higher a liquid’s temperature, the faster it will evaporate.

• The lower a liquid’s pressure, the faster the liquid will evaporate.

• The more area of liquid that is exposed to air, the faster the liquid will evaporate.

• The more circulation of air above a liquid, the faster the liquid will evaporate.

Boiling Point• Increased pressure on a liquid raises the liquid’s boiling point.

• Decreased pressure on a liquid lowers the liquid’s boiling point.

Page 103: AT - 4 - Zohowinlearning.zohosites.com › files › PDF › Applied... · The use of materials in this manual does not imply any specific results when WIN materials are used with

Applied Technology • 103

REFERENCE

BIBLIOGRAPHY

Adkinson, S., & Fleer, M. (Eds.). (1995). Science with reason. London: Hodder andStoughton Educational.

American Association for the Advancement of Science (1993). Benchmarks for scienceliteracy: A project 2061 report. New York: Oxford University Press.

American Association for the Advancement of Science (1990). Science for all Americans:A project 2061 report on literacy goals in science, mathematics, and technology. New York:Oxford University Press.

Bransford, J., & Stein, B. (1984). The IDEAL problem solver: A guide for improvingthinking, learning, and creativity. New York: W. H. Freeman and Co.

Ohio Department of Education (1994). Ohio’s competency-based science model: Scientificliteracy for the 21st century. Columbus, OH: State Board of Education.

National Council of Teachers of Mathematics (1993). Curriculum and evaluationstandards for school mathematics. Reston, VA: NCTM.

National Research Council (1996). National science educational standards. Washington,DC: National Academy Press.

The Secretary’s Commission on Achieving Necessary Skills (1992). Learning a living:A blueprint for high performance: A SCANS report for America 2000. Washington, DC:U.S. Department of Labor.

Page 104: AT - 4 - Zohowinlearning.zohosites.com › files › PDF › Applied... · The use of materials in this manual does not imply any specific results when WIN materials are used with

104 • Applied Technology

ANSWERS TO POP QUIZ QUESTIONS

Page 10 — The metal club in EdWIN’s hand is a conductor.

Page 33 — Resistance – opposition to a force.

Page 71 — The rate at which electricity flows is measured in amperes oramps.

Page 105: AT - 4 - Zohowinlearning.zohosites.com › files › PDF › Applied... · The use of materials in this manual does not imply any specific results when WIN materials are used with
Page 106: AT - 4 - Zohowinlearning.zohosites.com › files › PDF › Applied... · The use of materials in this manual does not imply any specific results when WIN materials are used with

Worldwide Interactive Network, Inc.1000 Waterford Place Kingston, TN 37763

Toll-free 888.717.9461Fax 865.717.9461 www.w-win.com

WIN Career Readiness Courseware - ©2008 Worldwide Interactive Network, Inc. All rights reserved.